Exos sympas MP(*)

Un problème, une question, un nouveau théorème ?

Messages : 0

Inscription : 18 avr. 2020 14:15

Profil de l'utilisateur : Élève de lycée

Re: Exos sympas MP(*)

Message par versionpatch » 06 mars 2021 20:48

Une belle preuve, beaucoup plus constructive que la mienne. Voici ma solution. On suppose que $rg(u,v,w) = 2$, que $(u,v)$ est $\mathbb{R}$ libre (le cas de rang $1$ se traite comme l'a fait Mourien) et on note $Z = \mathbb{Z}u + \mathbb{Z}v + \mathbb{Z}w$.

On munit $\mathbb{R}^{2}$ de la norme infinie sur la base $(u,v)$. En ramenant les $kw, k \in \mathbb{Z}^{*}$ par des translation entieres par u et v, on montre que $B(0,1)$ contient une infinite d'élements distincts de $Z$ et par compacité et le fait que $Z$ est stable par différence, $Z$ contient des élements non nuls de normes arbitrairement petits.

Soit $(z_{n})$ une suite d'élements dans $Z$ telle que $0 < ||z_{n}|| < 2^{-n}$ et $\frac{z_{n}}{||z_{n}||}$ converge vers $a \in S^{1}(0,1)$. Soit $\varepsilon > 0$ ,$y \in \mathbb{R}^{*}$,$n \in \mathbb{N}$ tel que $||z_{n}|| < \frac{\varepsilon}{2}$ et $||\frac{z_{n}}{||z_{n}||} - a|| < \frac{\varepsilon}{2|y|}$ et $K = \left[\frac{y}{||z_{n}||}\right]$. On a donc $$ ||Kz_{n} - ya|| \leq ||Kz_{n} - \frac{y}{||z_{n}||}z_{n} || + |y| \times ||\frac{z_{n}}{||z_{n}||} - a || < \varepsilon $$En fait, il n'est pas nécessaire de traiter le cas de rang $1$ à part, l'énoncé se géneralise à $(w_{1},...,w_{k+1})$ est $\mathbb{Q}$ libre dans $\mathbb{R}^{k} \implies$ L'adhérence de $\sum \mathbb{Z}w_{i}$ contient une droite. Il suffit de refaire la même preuve en supposant que $(w_{1},...,w_{r})$ est une base de l'espace engendré par ces $k+1$ vecteurs et travailler dans cet espace avec la famille $(w_{1},...,w_{r+1})$.
2018/2020 : MPSI/MP*
X2020

Messages : 3

Inscription : 22 mars 2020 15:08

Profil de l'utilisateur : Élève de lycée

Re: Exos sympas MP(*)

Message par Mourien » 06 mars 2021 22:07

Merci pour cette preuve ! La compacité est un outil très élégant ici !

Il reste celui-là :
Calli a écrit :
06 mars 2021 11:31
Moi aussi je propose un exercice :
Expliciter (par une formule) une famille libre indénombrable du $\mathbb Q$ espace vectoriel $\mathbb R$
Une indication ?
PCSI ; MP* ; ENS de Lyon

Messages : 4

Inscription : 13 oct. 2019 16:35

Profil de l'utilisateur : Élève de lycée

Re: Exos sympas MP(*)

Message par autobox » 07 mars 2021 19:01

Mourien a écrit :
06 mars 2021 22:07
Merci pour cette preuve ! La compacité est un outil très élégant ici !

Il reste celui-là :
Calli a écrit :
06 mars 2021 11:31
Moi aussi je propose un exercice :
Expliciter (par une formule) une famille libre indénombrable du $\mathbb Q$ espace vectoriel $\mathbb R$
Une indication ?
As-tu essayé de voir si ça fonctionne avec $ \left\{e^x \,|\, x\in M_3\right\} $, où $ M_3 $ est l'ensemble des réels dont l'écriture en base 3 ne possède aucun 2 (en gros, les centres des intervalles qu'on retire dans la construction de l'ensemble triadique de Cantor) ?

Ou peut-être, la famille $ \{1\}\cup\left\{\displaystyle\sum_{n=0}^\infty 2^{-\lfloor e^{nx}\rfloor} \,|\, x\in\mathbb{R}^\ast\right\} $ ?

En tout cas, ça m'a pas l'air simple cette histoire. Il est impossible d'exhiber une Q-base de Hamel de R sans l'axiome du choix, mais une famille libre ça m'a l'air possible.

V@J

Messages : 2812

Inscription : 22 janv. 2009 17:15

Profil de l'utilisateur : Élève de lycée

Re: Exos sympas MP(*)

Message par V@J » 10 mars 2021 08:17

Expliciter (par une formule) une famille libre indénombrable du Expliciter (par une formule) une famille libre indénombrable du Q espace vectoriel R
Ce n'est en fait pas trop méchant. On peut, par exemple, procéder comme suit : j'ai découpé mon raisonnement en plusieurs étapes, pour que l'on puisse voir l'une d'entre elles en se permettant de réfléchir à celles qui suivront.
SPOILER:
À tout sous-ensemble $ X $ de $ \mathbb{N} $ et tout entier $ k \geqslant 0 $, j'associe l'entier $ s_k(X) = \sum_{x=0}^k 2^x \mathbf{1}_{x \in X} $.
SPOILER:
À tout sous-ensemble $ X $ de $ \mathbb{N} $, j'associe maintenant le réel $ f(X) = 1 + \sum_{k \geqslant 0} 2^{-k^2 2^k - k s_k(X)} $.
SPOILER:
La famille $ \{f(X) \colon X \in \mathbb{R}\} $ est libre sur $ \mathbb{Q} $.
SPOILER:
En effet, s'il existe une famille finie (non vide) d'ensembles $ X_1,X_2,\ldots,X_n $ et des rationnels non nuls $ \lambda_1,\ldots,\lambda_n $ tels que $ \sum_i \lambda_i f(X_i) = 0 $, on commence par multiplier tous nos $ \lambda_i $ par le produit de leurs dénominateurs pour supposer qu'ils sont entiers, puis on considère un entier $ k $ tel que les nombres $ s_k(X_i) $ sont deux à deux distincts, et tel que $ \sum_i |\lambda_i| < 2^k $.
SPOILER:
On peut alors vérifier que $ 2^{k^2 2^k + k s_k(X_0)} $ est à distance $ < 1 $ d'un entier non nul, ce qui conclut.

Messages : 173

Inscription : 11 mars 2021 18:24

Profil de l'utilisateur : Élève de lycée

Re: Exos sympas MP(*)

Message par Contrexemple » 11 mars 2021 19:37

Bonsoir,

Je vous en propose 2 :

Soit $ n \in\mathbb N,n\geq 2 $ et $ P \in \mathbb Q[x], deg(P)\leq n-1 $ tel que $ \forall j \in [1,n] \cap \mathbb N,P(j) \in \mathbb Z $.
A-t-on $ \forall j \in \mathbb Z,P(j) \in \mathbb Z $ ?

Soit $ R \in \mathbb Q(x) $ tel que pour tout nombre premier $p$, $R(p) \in \mathbb Z$.
A-t-on $ R \in \mathbb Q[x] $ ?

Quelques remarques préliminaires, le deuxième est plus difficile que le premier.
Je donne une indication pour le deuxième si vous séchez.
SPOILER:
on pourra se servir d'un résultat sur la localisation des racines d'un polynôme
Bonne soirée.

Messages : 3

Inscription : 22 mars 2020 15:08

Profil de l'utilisateur : Élève de lycée

Re: Exos sympas MP(*)

Message par Mourien » 13 mars 2021 19:24

Pour la 1, c'est non.
SPOILER:
$ n=3 $ et $ P(x)=\dfrac 12 (x-1)(x-2) $
Je me penche sur la 2 !
PCSI ; MP* ; ENS de Lyon

Inversion

Re: Exos sympas MP(*)

Message par Inversion » 13 mars 2021 19:32

Mourien a écrit :
13 mars 2021 19:24
Pour la 1, c'est non.
SPOILER:
$ n=3 $ et $ P(x)=\dfrac 12 (x-1)(x-2) $
Je me penche sur la 2 !
Je ne comprends pas en quoi c'est un contre-exemple :?

Messages : 4

Inscription : 13 oct. 2019 16:35

Profil de l'utilisateur : Élève de lycée

Re: Exos sympas MP(*)

Message par autobox » 13 mars 2021 19:42

Inversion a écrit :
13 mars 2021 19:32
Mourien a écrit :
13 mars 2021 19:24
Pour la 1, c'est non.
SPOILER:
$ n=3 $ et $ P(x)=\dfrac 12 (x-1)(x-2) $
Je me penche sur la 2 !
Je ne comprends pas en quoi c'est un contre-exemple :?
Et ça ne risque pas de fonctionner puisque (n-1)(n-2) est toujours un entier pair :)

Messages : 3

Inscription : 22 mars 2020 15:08

Profil de l'utilisateur : Élève de lycée

Re: Exos sympas MP(*)

Message par Mourien » 13 mars 2021 19:46

En effet :oops:
PCSI ; MP* ; ENS de Lyon

Inversion

Re: Exos sympas MP(*)

Message par Inversion » 13 mars 2021 20:20

$ $Voici des indications pour la première si tu veux :
SPOILER:
Si je ne dis pas de bêtise, la réponse est oui
SPOILER:
Considérer la famille de polynômes $(P_k)$ définie par $P_0=1$ et pour $k \ge 1$, $P_k(X) = (X-1) \cdots (X-k) / k!$.
SPOILER:
Montrer que pour tout $k$ entier naturel on a $P_k(\mathbb{Z}) \subset \mathbb{Z}$
SPOILER:
La famille $(P_k)$ est une base de $R[X]$
SPOILER:
Conclure

Répondre